Line 3: Line 3:
 
'''1. (30 Points)'''
 
'''1. (30 Points)'''
  
Assume that <math>\mathbf{X}</math>  is a binomial distributed random variable with probability mass function (pmf) given by <math>p_{n}\left(k\right)=\left(\begin{array}{c}
+
Assume that <math class="inline">\mathbf{X}</math>  is a binomial distributed random variable with probability mass function (pmf) given by <math class="inline">p_{n}\left(k\right)=\left(\begin{array}{c}
 
n\\
 
n\\
 
k
 
k
\end{array}\right)p^{k}\left(1-p\right)^{n-k}\;,\qquad k=0,1,2,\cdots,n,</math><math> where <math>0<p<1</math> .
+
\end{array}\right)p^{k}\left(1-p\right)^{n-k}\;,\qquad k=0,1,2,\cdots,n,</math><math class="inline"> where <math class="inline">0<p<1</math> .
  
 
'''(a)'''
 
'''(a)'''
  
Find the characteristic function of <math>\mathbf{X}</math> . (You must show how you derive the characteristic function.)
+
Find the characteristic function of <math class="inline">\mathbf{X}</math> . (You must show how you derive the characteristic function.)
  
<math>\Phi_{\mathbf{X}}\left(\omega\right)=E\left[e^{i\omega\mathbf{X}}\right]=\sum_{k=0}^{n}e^{i\omega k}\cdot\left(\begin{array}{c}
+
<math class="inline">\Phi_{\mathbf{X}}\left(\omega\right)=E\left[e^{i\omega\mathbf{X}}\right]=\sum_{k=0}^{n}e^{i\omega k}\cdot\left(\begin{array}{c}
 
n\\
 
n\\
 
k
 
k
Line 18: Line 18:
 
n\\
 
n\\
 
k
 
k
\end{array}\right)\left(p\cdot e^{i\omega}\right)^{k}\left(1-p\right)^{n-k}</math><math>=\left(p\cdot e^{i\omega}+1-p\right)^{n}.</math>  
+
\end{array}\right)\left(p\cdot e^{i\omega}\right)^{k}\left(1-p\right)^{n-k}</math><math class="inline">=\left(p\cdot e^{i\omega}+1-p\right)^{n}.</math>  
  
 
'''(b)'''  
 
'''(b)'''  
  
Compute the standard deviation of <math>\mathbf{X}</math> .
+
Compute the standard deviation of <math class="inline">\mathbf{X}</math> .
  
<math>E\left[\mathbf{X}\right]=\frac{d}{di\omega}\Phi_{\mathbf{X}}\left(\omega\right)\Bigl|_{i\omega=0}=n\left(pe^{i\omega}+1-p\right)^{n-1}\cdot pe^{i\omega}\Bigl|_{i\omega=0}=np.</math>  
+
<math class="inline">E\left[\mathbf{X}\right]=\frac{d}{di\omega}\Phi_{\mathbf{X}}\left(\omega\right)\Bigl|_{i\omega=0}=n\left(pe^{i\omega}+1-p\right)^{n-1}\cdot pe^{i\omega}\Bigl|_{i\omega=0}=np.</math>  
  
<math>E\left[\mathbf{X}^{2}\right]=\frac{d^{2}}{d\left(i\omega\right)^{2}}\Phi_{\mathbf{X}}\left(\omega\right)\Bigl|_{i\omega=0}=\frac{d}{di\omega}npe^{i\omega}\left(pe^{i\omega}+1-p\right)^{n-1}\Bigl|_{i\omega=0}</math><math>=np\left[\frac{d}{di\omega}e^{i\omega}\left(pe^{i\omega}+1-p\right)^{n-1}\Bigl|_{i\omega=0}\right]</math><math>=np\left[e^{i\omega}\left(pe^{i\omega}+1-p\right)^{n-1}+e^{i\omega}(n-1)\left(pe^{i\omega}+1-p\right)^{n-2}\cdot pe^{i\omega}\Bigl|_{i\omega=0}\right]</math><math>=np\left[1+\left(n-1\right)p\right]=np+n\left(n-1\right)p^{2}.</math>  
+
<math class="inline">E\left[\mathbf{X}^{2}\right]=\frac{d^{2}}{d\left(i\omega\right)^{2}}\Phi_{\mathbf{X}}\left(\omega\right)\Bigl|_{i\omega=0}=\frac{d}{di\omega}npe^{i\omega}\left(pe^{i\omega}+1-p\right)^{n-1}\Bigl|_{i\omega=0}</math><math class="inline">=np\left[\frac{d}{di\omega}e^{i\omega}\left(pe^{i\omega}+1-p\right)^{n-1}\Bigl|_{i\omega=0}\right]</math><math class="inline">=np\left[e^{i\omega}\left(pe^{i\omega}+1-p\right)^{n-1}+e^{i\omega}(n-1)\left(pe^{i\omega}+1-p\right)^{n-2}\cdot pe^{i\omega}\Bigl|_{i\omega=0}\right]</math><math class="inline">=np\left[1+\left(n-1\right)p\right]=np+n\left(n-1\right)p^{2}.</math>  
  
<math>Var\left[\mathbf{X}\right]=E\left[\mathbf{X}^{2}\right]-\left(E\left[\mathbf{X}\right]\right)^{2}=np+n\left(n-1\right)p^{2}-\left(np\right)^{2}=np+n^{2}p^{2}-np^{2}-n^{2}p^{2}=np\left(1-p\right).</math>  
+
<math class="inline">Var\left[\mathbf{X}\right]=E\left[\mathbf{X}^{2}\right]-\left(E\left[\mathbf{X}\right]\right)^{2}=np+n\left(n-1\right)p^{2}-\left(np\right)^{2}=np+n^{2}p^{2}-np^{2}-n^{2}p^{2}=np\left(1-p\right).</math>  
  
<math>\therefore\sigma_{\mathbf{X}}=\sqrt{np\left(1-p\right)}.</math>  
+
<math class="inline">\therefore\sigma_{\mathbf{X}}=\sqrt{np\left(1-p\right)}.</math>  
  
 
'''(c)'''
 
'''(c)'''
  
Find the value or values of <math>k</math>  for which <math>p_{n}\left(k\right)</math>  is maximum, and express the answer in terms of <math>p</math>  and <math>n</math> . Give the most complete answer to this question that you can.
+
Find the value or values of <math class="inline">k</math>  for which <math class="inline">p_{n}\left(k\right)</math>  is maximum, and express the answer in terms of <math class="inline">p</math>  and <math class="inline">n</math> . Give the most complete answer to this question that you can.
  
<math>\frac{p_{n}\left(k\right)}{p_{n}\left(k-1\right)}=\frac{\left(\begin{array}{c}
+
<math class="inline">\frac{p_{n}\left(k\right)}{p_{n}\left(k-1\right)}=\frac{\left(\begin{array}{c}
 
n\\
 
n\\
 
k
 
k
Line 42: Line 42:
 
n\\
 
n\\
 
k-1
 
k-1
\end{array}\right)p^{k-1}\left(1-p\right)^{n-k+1}}=\frac{\frac{n!}{\left(n-k\right)!k!}p^{k}\left(1-p\right)^{n-k}}{\frac{n!}{\left(n-k+1\right)!\left(k-1\right)!}p^{k-1}\left(1-p\right)^{n-k+1}}</math><math>=\frac{n!}{\left(n-k\right)!k!}\cdot\frac{\left(n-k+1\right)!\left(k-1\right)!}{n!}\cdot\frac{p}{1-p}=\frac{n-k+1}{k}\cdot\frac{p}{1-p}.</math>
+
\end{array}\right)p^{k-1}\left(1-p\right)^{n-k+1}}=\frac{\frac{n!}{\left(n-k\right)!k!}p^{k}\left(1-p\right)^{n-k}}{\frac{n!}{\left(n-k+1\right)!\left(k-1\right)!}p^{k-1}\left(1-p\right)^{n-k+1}}</math><math class="inline">=\frac{n!}{\left(n-k\right)!k!}\cdot\frac{\left(n-k+1\right)!\left(k-1\right)!}{n!}\cdot\frac{p}{1-p}=\frac{n-k+1}{k}\cdot\frac{p}{1-p}.</math>
  
<math>\frac{p_{n}\left(k\right)}{p_{n}\left(k-1\right)}</math>  is monotonically descreasing function of <math>k</math> . Hence, <math>p_{n}\left(k\right)</math>  is maximized by the largest <math>k</math>  such that <math>\frac{p_{n}\left(k\right)}{p_{n}\left(k-1\right)}\geq1</math> .<math>\left(n-k+1\right)p\geq k\left(1-p\right)\Longrightarrow np-kp+p\geq k-kp\Longrightarrow\left(n+1\right)p\geq k</math>. <math>k</math>  should be the integer. Thus, <math>k=\left\lfloor \left(n+1\right)p\right\rfloor</math>  maximize <math>p_{n}\left(k\right)</math> . If <math>p_{n}\left(k\right)=p_{n}\left(k-1\right)</math> , both <math>k=\left(n+1\right)p</math>  and <math>k=\left(n+1\right)p-1</math>  maximize <math>p_{n}\left(k\right)</math> .
+
<math class="inline">\frac{p_{n}\left(k\right)}{p_{n}\left(k-1\right)}</math>  is monotonically descreasing function of <math class="inline">k</math> . Hence, <math class="inline">p_{n}\left(k\right)</math>  is maximized by the largest <math class="inline">k</math>  such that <math class="inline">\frac{p_{n}\left(k\right)}{p_{n}\left(k-1\right)}\geq1</math> .<math class="inline">\left(n-k+1\right)p\geq k\left(1-p\right)\Longrightarrow np-kp+p\geq k-kp\Longrightarrow\left(n+1\right)p\geq k</math>. <math class="inline">k</math>  should be the integer. Thus, <math class="inline">k=\left\lfloor \left(n+1\right)p\right\rfloor</math>  maximize <math class="inline">p_{n}\left(k\right)</math> . If <math class="inline">p_{n}\left(k\right)=p_{n}\left(k-1\right)</math> , both <math class="inline">k=\left(n+1\right)p</math>  and <math class="inline">k=\left(n+1\right)p-1</math>  maximize <math class="inline">p_{n}\left(k\right)</math> .
  
 
'''2. (30 Points)'''
 
'''2. (30 Points)'''
  
Let <math>\mathbf{X}_{1},\mathbf{X}_{2},\cdots,\mathbf{X}_{n},\cdots</math>  be a sequence of binomially distributed random variables, with <math>\mathbf{X}_{n}</math>  having probability mass function <math>p_{n}\left(k\right)=\left(\begin{array}{c}
+
Let <math class="inline">\mathbf{X}_{1},\mathbf{X}_{2},\cdots,\mathbf{X}_{n},\cdots</math>  be a sequence of binomially distributed random variables, with <math class="inline">\mathbf{X}_{n}</math>  having probability mass function <math class="inline">p_{n}\left(k\right)=\left(\begin{array}{c}
 
n\\
 
n\\
 
k
 
k
\end{array}\right)p_{n}^{k}\left(1-p_{n}\right)^{n-k}\;,\qquad k=0,1,2,\cdots,n,</math> where <math>0<p_{n}<1</math>  for all <math>n=1,2,3,\cdots</math> . Show that if <math>np_{n}\rightarrow\lambda\text{ as }n\rightarrow\infty,</math> then the random sequence <math>\mathbf{X}_{1},\mathbf{X}_{2},\cdots,\mathbf{X}_{n},\cdots</math>  converges in distribution to a Poisson random variable having mean <math>\lambda</math> .
+
\end{array}\right)p_{n}^{k}\left(1-p_{n}\right)^{n-k}\;,\qquad k=0,1,2,\cdots,n,</math> where <math class="inline">0<p_{n}<1</math>  for all <math class="inline">n=1,2,3,\cdots</math> . Show that if <math class="inline">np_{n}\rightarrow\lambda\text{ as }n\rightarrow\infty,</math> then the random sequence <math class="inline">\mathbf{X}_{1},\mathbf{X}_{2},\cdots,\mathbf{X}_{n},\cdots</math>  converges in distribution to a Poisson random variable having mean <math class="inline">\lambda</math> .
  
<math>\Phi_{\mathbf{X}_{n}}\left(\omega\right)=E\left[e^{i\omega\mathbf{X}_{n}}\right]=\sum_{k=0}^{n}e^{i\omega k}\left(\begin{array}{c}
+
<math class="inline">\Phi_{\mathbf{X}_{n}}\left(\omega\right)=E\left[e^{i\omega\mathbf{X}_{n}}\right]=\sum_{k=0}^{n}e^{i\omega k}\left(\begin{array}{c}
 
n\\
 
n\\
 
k
 
k
Line 59: Line 59:
 
n\\
 
n\\
 
k
 
k
\end{array}\right)\left(e^{i\omega}p_{n}\right)^{k}\left(1-p_{n}\right)^{n-k}</math><math>=\left(e^{i\omega}p_{n}+1-p_{n}\right)^{n}=\left(1-p_{n}\left(e^{i\omega}-1\right)\right)^{n}.</math>
+
\end{array}\right)\left(e^{i\omega}p_{n}\right)^{k}\left(1-p_{n}\right)^{n-k}</math><math class="inline">=\left(e^{i\omega}p_{n}+1-p_{n}\right)^{n}=\left(1-p_{n}\left(e^{i\omega}-1\right)\right)^{n}.</math>
  
Since <math>np_{n}\rightarrow\lambda</math>  as <math>n\rightarrow\infty</math> , <math>p_{n}\rightarrow\lambda/n</math> . <math>\lim_{n\rightarrow\infty}\Phi_{\mathbf{X}_{n}}\left(\omega\right)=\lim_{n\rightarrow\infty}\left(1-p_{n}\left(e^{i\omega}-1\right)\right)^{n}=\lim_{n\rightarrow\infty}\left(1-\frac{\lambda}{n}\left(e^{i\omega}-1\right)\right)^{n}=e^{\lambda\left(e^{i\omega}-1\right)}.</math>  
+
Since <math class="inline">np_{n}\rightarrow\lambda</math>  as <math class="inline">n\rightarrow\infty</math> , <math class="inline">p_{n}\rightarrow\lambda/n</math> . <math class="inline">\lim_{n\rightarrow\infty}\Phi_{\mathbf{X}_{n}}\left(\omega\right)=\lim_{n\rightarrow\infty}\left(1-p_{n}\left(e^{i\omega}-1\right)\right)^{n}=\lim_{n\rightarrow\infty}\left(1-\frac{\lambda}{n}\left(e^{i\omega}-1\right)\right)^{n}=e^{\lambda\left(e^{i\omega}-1\right)}.</math>  
  
<math>\because\lim_{n\rightarrow\infty}\left(1+\frac{x}{n}\right)^{n}=e^{x}.</math>  
+
<math class="inline">\because\lim_{n\rightarrow\infty}\left(1+\frac{x}{n}\right)^{n}=e^{x}.</math>  
  
<math>e^{\lambda\left(e^{i\omega}-1\right)}</math>  is the characteristic function of a Poisson random variable with mean <math>\lambda</math> . Thus, as <math>n\rightarrow\infty</math> , <math>\mathbf{X}_{n}</math>  converges in distribution to a Poisson random variable with mean <math>\lambda</math> .
+
<math class="inline">e^{\lambda\left(e^{i\omega}-1\right)}</math>  is the characteristic function of a Poisson random variable with mean <math class="inline">\lambda</math> . Thus, as <math class="inline">n\rightarrow\infty</math> , <math class="inline">\mathbf{X}_{n}</math>  converges in distribution to a Poisson random variable with mean <math class="inline">\lambda</math> .
  
 
'''3. (40 Points)'''
 
'''3. (40 Points)'''
  
Consider a homogeneous Poisson point process with rate <math>\lambda</math>  and points (event occurrence times) <math>\mathbf{T}_{1},\mathbf{T}_{2},\cdots,\mathbf{T}_{n},\cdots</math> .
+
Consider a homogeneous Poisson point process with rate <math class="inline">\lambda</math>  and points (event occurrence times) <math class="inline">\mathbf{T}_{1},\mathbf{T}_{2},\cdots,\mathbf{T}_{n},\cdots</math> .
  
 
'''(a)'''
 
'''(a)'''
  
Derive the pdf <math>f_{k}\left(t\right)</math>  of the <math>k</math> -th point <math>\mathbf{T}_{k}</math>  for arbitrary <math>k</math> .
+
Derive the pdf <math class="inline">f_{k}\left(t\right)</math>  of the <math class="inline">k</math> -th point <math class="inline">\mathbf{T}_{k}</math>  for arbitrary <math class="inline">k</math> .
  
 
• The cdf is  
 
• The cdf is  
<math>F_{\mathbf{T}_{k}}\left(t\right)=P\left(\mathbf{T}_{k}\leq t\right)=P\left(\text{at least }k\text{ points within }t\right)=\sum_{j=k}^{\infty}\frac{e^{-\lambda t}\cdot\left(\lambda t\right)^{j}}{j!}</math><math>=1-P\left(\mathbf{T}_{k}>t\right)=1-P\left(\text{less than }k\text{ points within }t\right)=1-\sum_{j=0}^{k-1}\frac{e^{-\lambda t}\cdot\left(\lambda t\right)^{j}}{j!}.</math>
+
<math class="inline">F_{\mathbf{T}_{k}}\left(t\right)=P\left(\mathbf{T}_{k}\leq t\right)=P\left(\text{at least }k\text{ points within }t\right)=\sum_{j=k}^{\infty}\frac{e^{-\lambda t}\cdot\left(\lambda t\right)^{j}}{j!}</math><math class="inline">=1-P\left(\mathbf{T}_{k}>t\right)=1-P\left(\text{less than }k\text{ points within }t\right)=1-\sum_{j=0}^{k-1}\frac{e^{-\lambda t}\cdot\left(\lambda t\right)^{j}}{j!}.</math>
  
 
• The pdf by differentiating the cdf  
 
• The pdf by differentiating the cdf  
<math>isf_{\mathbf{T}_{k}}\left(t\right)=\frac{dF_{\mathbf{T}_{k}}}{dt}=-\sum_{j=0}^{k-1}\frac{\left(-\lambda\right)e^{-\lambda t}\cdot\left(\lambda t\right)^{j}}{j!}-\sum_{j=0}^{k-1}\frac{e^{-\lambda t}\cdot j\left(\lambda t\right)^{j-1}\cdot\lambda}{j!}</math><math>=\lambda e^{-\lambda t}\sum_{j=0}^{k-1}\frac{\left(\lambda t\right)^{j}}{j!}-\lambda e^{-\lambda t}\sum_{j=1}^{k-1}\frac{j\left(\lambda t\right)^{j-1}}{j!}=\lambda e^{-\lambda t}\sum_{j=0}^{k-1}\frac{\left(\lambda t\right)^{j}}{j!}-\lambda e^{-\lambda t}\sum_{j=1}^{k-1}\frac{\left(\lambda t\right)^{j-1}}{\left(j-1\right)!}</math><math>=\lambda e^{-\lambda t}\sum_{j=0}^{k-1}\frac{\left(\lambda t\right)^{j}}{j!}-\lambda e^{-\lambda t}\sum_{j=0}^{k-2}\frac{\left(\lambda t\right)^{j}}{j!}=\lambda e^{-\lambda t}\frac{\left(\lambda t\right)^{k-1}}{\left(k-1\right)!}.</math>  
+
<math class="inline">isf_{\mathbf{T}_{k}}\left(t\right)=\frac{dF_{\mathbf{T}_{k}}}{dt}=-\sum_{j=0}^{k-1}\frac{\left(-\lambda\right)e^{-\lambda t}\cdot\left(\lambda t\right)^{j}}{j!}-\sum_{j=0}^{k-1}\frac{e^{-\lambda t}\cdot j\left(\lambda t\right)^{j-1}\cdot\lambda}{j!}</math><math class="inline">=\lambda e^{-\lambda t}\sum_{j=0}^{k-1}\frac{\left(\lambda t\right)^{j}}{j!}-\lambda e^{-\lambda t}\sum_{j=1}^{k-1}\frac{j\left(\lambda t\right)^{j-1}}{j!}=\lambda e^{-\lambda t}\sum_{j=0}^{k-1}\frac{\left(\lambda t\right)^{j}}{j!}-\lambda e^{-\lambda t}\sum_{j=1}^{k-1}\frac{\left(\lambda t\right)^{j-1}}{\left(j-1\right)!}</math><math class="inline">=\lambda e^{-\lambda t}\sum_{j=0}^{k-1}\frac{\left(\lambda t\right)^{j}}{j!}-\lambda e^{-\lambda t}\sum_{j=0}^{k-2}\frac{\left(\lambda t\right)^{j}}{j!}=\lambda e^{-\lambda t}\frac{\left(\lambda t\right)^{k-1}}{\left(k-1\right)!}.</math>  
  
 
– This is Erlang distribution.
 
– This is Erlang distribution.
Line 85: Line 85:
 
'''(b)'''
 
'''(b)'''
  
What kind of distribution does <math>\mathbf{T}_{1}</math>  have?
+
What kind of distribution does <math class="inline">\mathbf{T}_{1}</math>  have?
  
• If <math>k=1</math> , then <math>f_{\mathbf{T}_{1}}\left(t\right)=\lambda e^{-\lambda t}</math> . Thus <math>\mathbf{T}_{1}</math>  is a exponential random variable with parameter <math>\lambda</math> .
+
• If <math class="inline">k=1</math> , then <math class="inline">f_{\mathbf{T}_{1}}\left(t\right)=\lambda e^{-\lambda t}</math> . Thus <math class="inline">\mathbf{T}_{1}</math>  is a exponential random variable with parameter <math class="inline">\lambda</math> .
  
 
'''(c)'''
 
'''(c)'''
  
What is the conditional pdf of <math>\mathbf{T}_{k}</math>  given <math>\mathbf{T}_{k-1}=t_{0}</math> , where <math>t_{0}>0</math> ? (You can give the answer without derivation if you know it.)
+
What is the conditional pdf of <math class="inline">\mathbf{T}_{k}</math>  given <math class="inline">\mathbf{T}_{k-1}=t_{0}</math> , where <math class="inline">t_{0}>0</math> ? (You can give the answer without derivation if you know it.)
  
 
• The conditional cdf is
 
• The conditional cdf is
  
<math>F_{\mathbf{T}_{k}}\left(t_{k}|\mathbf{T}_{k-1}=t_{0}\right)=P\left(\mathbf{T}_{k}\leq t_{k}|\mathbf{T}_{k-1}=t_{0}\right)=P\left(N\left(t_{k},t_{0}\right)\geq1\right)</math><math>=1-P\left(N\left(t_{k},t_{0}\right)=0\right)=1-e^{-\lambda\left(t_{k}-t_{0}\right)}.</math>  
+
<math class="inline">F_{\mathbf{T}_{k}}\left(t_{k}|\mathbf{T}_{k-1}=t_{0}\right)=P\left(\mathbf{T}_{k}\leq t_{k}|\mathbf{T}_{k-1}=t_{0}\right)=P\left(N\left(t_{k},t_{0}\right)\geq1\right)</math><math class="inline">=1-P\left(N\left(t_{k},t_{0}\right)=0\right)=1-e^{-\lambda\left(t_{k}-t_{0}\right)}.</math>  
  
 
• The conditional pdf by differentiating the conditional cdf is
 
• The conditional pdf by differentiating the conditional cdf is
  
<math>\therefore f_{\mathbf{T}_{k}}\left(t_{k}|\mathbf{T}_{k-1}=t_{0}\right)=\lambda e^{-\lambda\left(t_{k}-t_{0}\right)}.</math>  
+
<math class="inline">\therefore f_{\mathbf{T}_{k}}\left(t_{k}|\mathbf{T}_{k-1}=t_{0}\right)=\lambda e^{-\lambda\left(t_{k}-t_{0}\right)}.</math>  
  
 
'''(d)'''
 
'''(d)'''
  
Suppose you have a random number generator that produces independent, identically distributed (i.i.d. ) random variables <math>\mathbf{X}_{1},\mathbf{X}_{2},\cdots,\mathbf{X}_{n},\cdots</math>  that are uniformaly distributed on the interval <math>\left(0,1\right)</math> . Explain how you could use these to simulate the Poisson points <math>\mathbf{T}_{1},\mathbf{T}_{2},\cdots,\mathbf{T}_{n},\cdots</math>  describe above. Provide as complete an explanation as possible.
+
Suppose you have a random number generator that produces independent, identically distributed (i.i.d. ) random variables <math class="inline">\mathbf{X}_{1},\mathbf{X}_{2},\cdots,\mathbf{X}_{n},\cdots</math>  that are uniformaly distributed on the interval <math class="inline">\left(0,1\right)</math> . Explain how you could use these to simulate the Poisson points <math class="inline">\mathbf{T}_{1},\mathbf{T}_{2},\cdots,\mathbf{T}_{n},\cdots</math>  describe above. Provide as complete an explanation as possible.
  
 
• This problem is similar to [[ECE600 QE 2003 August|QE 2003 August]] Problem 2.  
 
• This problem is similar to [[ECE600 QE 2003 August|QE 2003 August]] Problem 2.  

Revision as of 07:32, 1 December 2010

7.10 QE 2005 August

1. (30 Points)

Assume that $ \mathbf{X} $ is a binomial distributed random variable with probability mass function (pmf) given by $ p_{n}\left(k\right)=\left(\begin{array}{c} n\\ k \end{array}\right)p^{k}\left(1-p\right)^{n-k}\;,\qquad k=0,1,2,\cdots,n, $$ where <math class="inline">0<p<1 $ .

(a)

Find the characteristic function of $ \mathbf{X} $ . (You must show how you derive the characteristic function.)

$ \Phi_{\mathbf{X}}\left(\omega\right)=E\left[e^{i\omega\mathbf{X}}\right]=\sum_{k=0}^{n}e^{i\omega k}\cdot\left(\begin{array}{c} n\\ k \end{array}\right)p^{k}\left(1-p\right)^{n-k}=\sum_{k=0}^{n}\left(\begin{array}{c} n\\ k \end{array}\right)\left(p\cdot e^{i\omega}\right)^{k}\left(1-p\right)^{n-k} $$ =\left(p\cdot e^{i\omega}+1-p\right)^{n}. $

(b)

Compute the standard deviation of $ \mathbf{X} $ .

$ E\left[\mathbf{X}\right]=\frac{d}{di\omega}\Phi_{\mathbf{X}}\left(\omega\right)\Bigl|_{i\omega=0}=n\left(pe^{i\omega}+1-p\right)^{n-1}\cdot pe^{i\omega}\Bigl|_{i\omega=0}=np. $

$ E\left[\mathbf{X}^{2}\right]=\frac{d^{2}}{d\left(i\omega\right)^{2}}\Phi_{\mathbf{X}}\left(\omega\right)\Bigl|_{i\omega=0}=\frac{d}{di\omega}npe^{i\omega}\left(pe^{i\omega}+1-p\right)^{n-1}\Bigl|_{i\omega=0} $$ =np\left[\frac{d}{di\omega}e^{i\omega}\left(pe^{i\omega}+1-p\right)^{n-1}\Bigl|_{i\omega=0}\right] $$ =np\left[e^{i\omega}\left(pe^{i\omega}+1-p\right)^{n-1}+e^{i\omega}(n-1)\left(pe^{i\omega}+1-p\right)^{n-2}\cdot pe^{i\omega}\Bigl|_{i\omega=0}\right] $$ =np\left[1+\left(n-1\right)p\right]=np+n\left(n-1\right)p^{2}. $

$ Var\left[\mathbf{X}\right]=E\left[\mathbf{X}^{2}\right]-\left(E\left[\mathbf{X}\right]\right)^{2}=np+n\left(n-1\right)p^{2}-\left(np\right)^{2}=np+n^{2}p^{2}-np^{2}-n^{2}p^{2}=np\left(1-p\right). $

$ \therefore\sigma_{\mathbf{X}}=\sqrt{np\left(1-p\right)}. $

(c)

Find the value or values of $ k $ for which $ p_{n}\left(k\right) $ is maximum, and express the answer in terms of $ p $ and $ n $ . Give the most complete answer to this question that you can.

$ \frac{p_{n}\left(k\right)}{p_{n}\left(k-1\right)}=\frac{\left(\begin{array}{c} n\\ k \end{array}\right)p^{k}\left(1-p\right)^{n-k}}{\left(\begin{array}{c} n\\ k-1 \end{array}\right)p^{k-1}\left(1-p\right)^{n-k+1}}=\frac{\frac{n!}{\left(n-k\right)!k!}p^{k}\left(1-p\right)^{n-k}}{\frac{n!}{\left(n-k+1\right)!\left(k-1\right)!}p^{k-1}\left(1-p\right)^{n-k+1}} $$ =\frac{n!}{\left(n-k\right)!k!}\cdot\frac{\left(n-k+1\right)!\left(k-1\right)!}{n!}\cdot\frac{p}{1-p}=\frac{n-k+1}{k}\cdot\frac{p}{1-p}. $

$ \frac{p_{n}\left(k\right)}{p_{n}\left(k-1\right)} $ is monotonically descreasing function of $ k $ . Hence, $ p_{n}\left(k\right) $ is maximized by the largest $ k $ such that $ \frac{p_{n}\left(k\right)}{p_{n}\left(k-1\right)}\geq1 $ .$ \left(n-k+1\right)p\geq k\left(1-p\right)\Longrightarrow np-kp+p\geq k-kp\Longrightarrow\left(n+1\right)p\geq k $. $ k $ should be the integer. Thus, $ k=\left\lfloor \left(n+1\right)p\right\rfloor $ maximize $ p_{n}\left(k\right) $ . If $ p_{n}\left(k\right)=p_{n}\left(k-1\right) $ , both $ k=\left(n+1\right)p $ and $ k=\left(n+1\right)p-1 $ maximize $ p_{n}\left(k\right) $ .

2. (30 Points)

Let $ \mathbf{X}_{1},\mathbf{X}_{2},\cdots,\mathbf{X}_{n},\cdots $ be a sequence of binomially distributed random variables, with $ \mathbf{X}_{n} $ having probability mass function $ p_{n}\left(k\right)=\left(\begin{array}{c} n\\ k \end{array}\right)p_{n}^{k}\left(1-p_{n}\right)^{n-k}\;,\qquad k=0,1,2,\cdots,n, $ where $ 0<p_{n}<1 $ for all $ n=1,2,3,\cdots $ . Show that if $ np_{n}\rightarrow\lambda\text{ as }n\rightarrow\infty, $ then the random sequence $ \mathbf{X}_{1},\mathbf{X}_{2},\cdots,\mathbf{X}_{n},\cdots $ converges in distribution to a Poisson random variable having mean $ \lambda $ .

$ \Phi_{\mathbf{X}_{n}}\left(\omega\right)=E\left[e^{i\omega\mathbf{X}_{n}}\right]=\sum_{k=0}^{n}e^{i\omega k}\left(\begin{array}{c} n\\ k \end{array}\right)p_{n}^{k}\left(1-p_{n}\right)^{n-k}=\sum_{k=0}^{n}\left(\begin{array}{c} n\\ k \end{array}\right)\left(e^{i\omega}p_{n}\right)^{k}\left(1-p_{n}\right)^{n-k} $$ =\left(e^{i\omega}p_{n}+1-p_{n}\right)^{n}=\left(1-p_{n}\left(e^{i\omega}-1\right)\right)^{n}. $

Since $ np_{n}\rightarrow\lambda $ as $ n\rightarrow\infty $ , $ p_{n}\rightarrow\lambda/n $ . $ \lim_{n\rightarrow\infty}\Phi_{\mathbf{X}_{n}}\left(\omega\right)=\lim_{n\rightarrow\infty}\left(1-p_{n}\left(e^{i\omega}-1\right)\right)^{n}=\lim_{n\rightarrow\infty}\left(1-\frac{\lambda}{n}\left(e^{i\omega}-1\right)\right)^{n}=e^{\lambda\left(e^{i\omega}-1\right)}. $

$ \because\lim_{n\rightarrow\infty}\left(1+\frac{x}{n}\right)^{n}=e^{x}. $

$ e^{\lambda\left(e^{i\omega}-1\right)} $ is the characteristic function of a Poisson random variable with mean $ \lambda $ . Thus, as $ n\rightarrow\infty $ , $ \mathbf{X}_{n} $ converges in distribution to a Poisson random variable with mean $ \lambda $ .

3. (40 Points)

Consider a homogeneous Poisson point process with rate $ \lambda $ and points (event occurrence times) $ \mathbf{T}_{1},\mathbf{T}_{2},\cdots,\mathbf{T}_{n},\cdots $ .

(a)

Derive the pdf $ f_{k}\left(t\right) $ of the $ k $ -th point $ \mathbf{T}_{k} $ for arbitrary $ k $ .

• The cdf is $ F_{\mathbf{T}_{k}}\left(t\right)=P\left(\mathbf{T}_{k}\leq t\right)=P\left(\text{at least }k\text{ points within }t\right)=\sum_{j=k}^{\infty}\frac{e^{-\lambda t}\cdot\left(\lambda t\right)^{j}}{j!} $$ =1-P\left(\mathbf{T}_{k}>t\right)=1-P\left(\text{less than }k\text{ points within }t\right)=1-\sum_{j=0}^{k-1}\frac{e^{-\lambda t}\cdot\left(\lambda t\right)^{j}}{j!}. $

• The pdf by differentiating the cdf $ isf_{\mathbf{T}_{k}}\left(t\right)=\frac{dF_{\mathbf{T}_{k}}}{dt}=-\sum_{j=0}^{k-1}\frac{\left(-\lambda\right)e^{-\lambda t}\cdot\left(\lambda t\right)^{j}}{j!}-\sum_{j=0}^{k-1}\frac{e^{-\lambda t}\cdot j\left(\lambda t\right)^{j-1}\cdot\lambda}{j!} $$ =\lambda e^{-\lambda t}\sum_{j=0}^{k-1}\frac{\left(\lambda t\right)^{j}}{j!}-\lambda e^{-\lambda t}\sum_{j=1}^{k-1}\frac{j\left(\lambda t\right)^{j-1}}{j!}=\lambda e^{-\lambda t}\sum_{j=0}^{k-1}\frac{\left(\lambda t\right)^{j}}{j!}-\lambda e^{-\lambda t}\sum_{j=1}^{k-1}\frac{\left(\lambda t\right)^{j-1}}{\left(j-1\right)!} $$ =\lambda e^{-\lambda t}\sum_{j=0}^{k-1}\frac{\left(\lambda t\right)^{j}}{j!}-\lambda e^{-\lambda t}\sum_{j=0}^{k-2}\frac{\left(\lambda t\right)^{j}}{j!}=\lambda e^{-\lambda t}\frac{\left(\lambda t\right)^{k-1}}{\left(k-1\right)!}. $

– This is Erlang distribution.

(b)

What kind of distribution does $ \mathbf{T}_{1} $ have?

• If $ k=1 $ , then $ f_{\mathbf{T}_{1}}\left(t\right)=\lambda e^{-\lambda t} $ . Thus $ \mathbf{T}_{1} $ is a exponential random variable with parameter $ \lambda $ .

(c)

What is the conditional pdf of $ \mathbf{T}_{k} $ given $ \mathbf{T}_{k-1}=t_{0} $ , where $ t_{0}>0 $ ? (You can give the answer without derivation if you know it.)

• The conditional cdf is

$ F_{\mathbf{T}_{k}}\left(t_{k}|\mathbf{T}_{k-1}=t_{0}\right)=P\left(\mathbf{T}_{k}\leq t_{k}|\mathbf{T}_{k-1}=t_{0}\right)=P\left(N\left(t_{k},t_{0}\right)\geq1\right) $$ =1-P\left(N\left(t_{k},t_{0}\right)=0\right)=1-e^{-\lambda\left(t_{k}-t_{0}\right)}. $

• The conditional pdf by differentiating the conditional cdf is

$ \therefore f_{\mathbf{T}_{k}}\left(t_{k}|\mathbf{T}_{k-1}=t_{0}\right)=\lambda e^{-\lambda\left(t_{k}-t_{0}\right)}. $

(d)

Suppose you have a random number generator that produces independent, identically distributed (i.i.d. ) random variables $ \mathbf{X}_{1},\mathbf{X}_{2},\cdots,\mathbf{X}_{n},\cdots $ that are uniformaly distributed on the interval $ \left(0,1\right) $ . Explain how you could use these to simulate the Poisson points $ \mathbf{T}_{1},\mathbf{T}_{2},\cdots,\mathbf{T}_{n},\cdots $ describe above. Provide as complete an explanation as possible.

• This problem is similar to QE 2003 August Problem 2.


Back to ECE600

Back to ECE 600 QE

Alumni Liaison

Recent Math PhD now doing a post-doctorate at UC Riverside.

Kuei-Nuan Lin